-518

Those of you who suffer from banner blindness may not have noticed the announcement:

Stack Overflow is changing the reputation scoring system to make the reputation earned from upvotes on questions equal to the reputation earned from upvotes on answers. Previously, upvotes on questions netted you only +5 reputation, whereas upvotes on answers netted you +10 reputation. Now, upvotes on posts will always be worth +10 reputation, regardless of whether that post is a question or an answer.

This change will not only take effect for contributions from here forward, but it will also be applied retroactively. A global reputation recalculation is pending and will be completed soon. Note that this recalculation will take the daily reputation cap into account. (The Mortarboard, Epic, Legendary, and Yearling badges will also be awarded, as they would have been earned under the new logic, but they are awarded nightly by a separate, background process, and thus may not show up at the same time as your recalculated reputation.)

The net effect will be to reward the contributions of users who have asked well-received questions, in recognition of the fact that useful, high-quality questions are just as valuable as high-quality answers. Asking a good question is difficult, takes practice, and is a meaningful contribution to the community at large, critical to our mission of building a high-quality library of questions and answers to practical programming problems.

No other changes are being made to the reputation system. Receiving downvotes on one of your questions will still reduce your reputation by the same amount as before. Casting downvotes on questions is still "free", in the sense that it does not reduce the reputation of the voter.

Why? You can find more detailed background information in my answer here, including a review of the history and an assessment of the facts that justify this change.

The biggest immediate change will be a substantial increase in user reputation scores, and thus an increase in the number of users having certain privileges.

As with far too many events of late, this change has been accompanied by a fair amount of drama. The proposed policy modifications were privately announced to community moderators by staff members some time back. Unfortunately, a less-than-ethical member of the larger Stack Exchange moderator community leaked this private communication to the public, and did so in a rather sensationalist way. I want to commend the Stack Exchange employees both for soliciting feedback from moderators and for listening to that feedback. I would ask that the community focus on the facts of the announcement and the policy as it has actually been implemented.

To that end, if you have questions, concerns, comments, or other feedback about this change, please post it as an answer here. You can also express your sentiments silently by voting. As always, we ask that you keep your feedback constructive and respectful. Feedback from those who cannot comply with this requirement is subject to removal.

137
  • 213
    We already have newbies complaining about these simple questions with hundreds of upvotes. Not very welcoming to double the score of these, forcing the poor new stackoverflowers to work even harder to catch up!
    – Patrice
    Nov 13, 2019 at 19:33
  • 278
    At this point I'm more and more surprised that we still come to SO. This might not last long, though. I feel so betrayed... Nov 13, 2019 at 19:38
  • 157
    I actually agree with this. Perhaps not the retroactive. But then again why should they not be same values as new upvotes. I find it way harder to write a question than a answer. And usually answers get far more upvotes than questions that can take up to hours to write good. A simple 5 min answer can get more points than it actually should in just a few minutes.
    – Andreas
    Nov 13, 2019 at 19:44
  • 334
    So valuing "lack of knowledge" the same as "actual knowledge" seemed like a good idea? I don't really care about the changes, but seems more like the goal is to get more questions, and so more content, not necessarily more good content in the form of good answers? This surely will benefit some of the top users, that early on have asked and answered their own questions on basic issues, with thousands of upvotes over the years.
    – adeneo
    Nov 13, 2019 at 19:55
  • 133
    @adeneo A knowledge base cannot exist without high-quality questions to solicit good answers. The reputation system isn't valuing knowledge at all. Rather, it is valuing your contributions to the Stack Overflow knowledge base (and thus serves as an indirect proxy for your familiarity and experience with this platform). The fundamental thought behind this policy is that good questions are a net positive contribution to the Stack Overflow knowledge base. Content that is not good can and should continue to be signaled by downvotes, whether questions or answers. Nov 13, 2019 at 20:03
  • 251
    @CodyGray - A fool can ask more questions than a wise man can answer, and at the end of the day SE still needs people that actually knows the answers to the questions, more than they do fools. My opinion is that answers should be valued higher than questions, and downvotes should count the same as upvotes, but what do I know, I've never really asked a single question?
    – adeneo
    Nov 13, 2019 at 20:09
  • 110
    Most of my reputation has come from question asking so far. But honestly I think the 5/10 split was fair: it's harder and less gratifying to write an answer, and the question-asker usually has some real-world benefit to getting their question answered as well. Questions definitely have a lot of value, but I think answers have even more value. This is not a horrible change, but I'm a little unsure whether it's the right one. Have there not been enough high-quality questions?
    – Stephen
    Nov 13, 2019 at 20:56
  • 87
    "The net effect will be to reward the contributions of everyone, including users who have bad questions that still exist on the site". Fixed that for you.
    – Herohtar
    Nov 13, 2019 at 21:33
  • 77
    Please don't blame users for not downvoting when downvoting is a privilege that you have to unlock and costs more rep than upvoting.
    – Unihedron
    Nov 13, 2019 at 21:40
  • 109
    @CodyGray The problem is, people don't downvote -- or even vote to close -- in cases where they should, and it seems to happen even less with all the recent push to be "nice". And people hand out upvotes just to "counter" a negative score on a question, which with this rep change rewards the poster even more than it previously did.
    – Herohtar
    Nov 13, 2019 at 21:43
  • 41
    But some of us only post answers and didn't just have our rep doubled, so we don't yet have close vote privileges, and now the relative weight of our downvotes just got cut in half...
    – manveti
    Nov 13, 2019 at 21:45
  • 212
    I just cannot figure out how can anyone that has spent more than five minutes moderating this site come to the conclusion that increasing reputation for the questions will actually result with more quality questions being asked. I am genuinely baffled.
    – Dalija Prasnikar Mod
    Nov 13, 2019 at 22:33
  • 61
    More questions = more page views. More important now than curating good answers. I fear stackexchange is swirling the bowl thanks to a radically different vision being forced down by the CORP. If you still care about your ranking, and you concentrate on giving good answers, consider this the same as cutting the value of all your answers in half. Nov 13, 2019 at 22:34
  • 117
    @CodyGray I'm sure you're caught in the middle of things here, but I couldn't help but notice who authored the blog introducing this change. It seems like SO in general is trying to continue on like it's business as usual with a blitz of blog posts and podcasts. There are much more important topics that need to be addressed and not swept under the rug, most importantly the breach of trust recent events have created.
    – jmoerdyk
    Nov 13, 2019 at 23:32
  • 157
    "OK, we succeeded in getting the more annoying mods and users to resign, but what about the remaining high-rep users who insist on downvoting and closing, so hitting ad stats? They're mostly highly skilled and experienced developers who are,politically aware'' ... "Hey! I know how we can really piss them off while looking good - we can't take their rep, but we can devalue it!" Nov 14, 2019 at 10:23

53 Answers 53

874

It looks like the question upvote weight is being doubled, but the downvote weight is not being increased in proportion.

I don't mind the change, but we should be rewarding good question askers, not just any question askers. Without the downvote weight also increasing, this change is more strongly incentivizing posting poor questions.

Why is the rep cost for having your question downvoted not also increasing?

To put things in perspective, a question which is downvoted 4 times and upvoted once will still be a net positive for the asker. And if you use Stack Overflow at all, you'll know questions that get downvoted in 4:1 ratio are generally garbage and should not be rewarded at all (they should be edited or closed/deleted).

31
  • 81
    Note that the rep cost for having your answer downvoted is still far less than the rep gained from having your answer upvoted. Does that policy make sense to you? And, if so, why should questions be any different? Nov 13, 2019 at 20:01
  • 31
    I also feel we'll be seeing a lot more "poor questions" out there. With this change you only need 1 upvote to balance 4 downvotes. After all you only need to get very few upvotes to make a poor question worthwhile.
    – Isac
    Nov 13, 2019 at 20:01
  • 157
    @CodyGray To be honest, no, it doesn't make sense to me. If it was my decision to make, the rep cost would be symmetric -10/+10 for both questions and answers.
    – wim
    Nov 13, 2019 at 20:02
  • 387
    Looks like we're optimizing for sand again Nov 13, 2019 at 20:02
  • 67
    I'm not entirely convinced that upping the Q-upvote rep will cause more poor questions. My impression is that vampires want to be spoon-fed. They don't care about rep. If someone is posting a question with the intention to farm rep, they're probably invested enough into the site to be less likely to post crap.
    – Mysticial
    Nov 13, 2019 at 20:08
  • 6
    I agree with your overall premise, but if a question gets downvoted 9 times and upvoted once, wouldn't that still put them at a net -8 rep? Nov 13, 2019 at 20:10
  • 8
    I suspect that question bans are based on upvote/downvote ratio rather than reputation, and that they're more important for keeping askers of bad questions away than reputation.
    – DavidW
    Nov 13, 2019 at 20:25
  • 11
    @JoeW If it's a low-traffic tag, probably won't even get closed (at least, not if the 3 votes to close policy isn't re-instated), to say nothing of deleted in a timely manner... Nov 13, 2019 at 21:08
  • 40
    Here is Jeff Atwoods take on increasing the weight of downvotes. Quote: Downvotes were always essentially cosmetic, with an extremely minor effect on reputation. Despite this, received downvotes are taken quite seriously by users. Almost too seriously.. Indeed, it seems that even if -4/+1 still has a positive reputation effect, it still upsets question askers. Garbage should be closed.
    – gerrit
    Nov 14, 2019 at 8:50
  • 5
    The perceived impact on reputation goes in the way of asking questions IMO. And a downvote when asking, especially if it looks like it hurts the small rep you have at beginning, can be very painful. Maybe questions should not impact reputation at all. Nov 14, 2019 at 9:40
  • 16
    I agree with @DenysSéguret, questions shouldn't impact reputation at all. Or at least not be valued as answers as this is comparing apples with pears. IMO there should be seperate counts, one for questions (if at all) and one for answers. Apart from that, the privileges system is problematic as well as it is coupled to reputation. It's about perception which makes it seem unfair. IMO privileges shouldn't be awarded based on reputation but rather earned based on participation and other factors. Leaving reputation for it's original purpose, to underline the quality of the answer / question.
    – user4864425
    Nov 14, 2019 at 10:58
  • 4
    Even with the downvote penalty doubled, this change still benefits the asker. An upvote/downvote pair used to net the asker 3 reputation. Even with your change, it would net them 6. (That's better than netting them 8, of course, but still a change for the worse.)
    – jpmc26
    Nov 14, 2019 at 21:38
  • 4
    @Xrylite Reputation doesn't exist on Meta sites, which is where all the disagreement over site policies happens. Regarding doubling the reputation lost from downvotes, see this related discussion from several years ago. Nov 14, 2019 at 21:52
  • 24
    @CodyGray Except on the Meta Stack Exchange which is where I'd like to use my downvotes to dissent the most. But hey, I guess I'll just sit at 99 there forever.
    – zero298
    Nov 15, 2019 at 19:19
  • 8
    I disagree. The best voted questions are simply the most commonly asked questions (no matter how basic and no matter how little effort is needed to find the answer). There is nothing wrong with this (IMO) and is the reason Stackoverflow is my go-to Q&A source and is really the concept SO was founded upon. SO didn't get where it is because people asked brilliantly crafted question after first spending 28 weeks researching the issue to avoid downvotes. It got where it is because it allowed common questions to exist (regardless of quality and solution-effort). My $.02 (and opinions). Nov 16, 2019 at 21:32
703

I'll ignore the problems of the decision being made behind walls and being contrary to what the community asked.

I'll ignore the fact that what we, as engineers, wanted, was more quality and less noise, not a bigger amount of trivial questions that maintained documentation (or sometimes similar previous questions) would better answer.

I'll address a deeper point of morality.

What made us answer, the promise behind the very word "reputation", which was described as "earned by convincing your peers that you know what you’re talking about", was that those points were a kind of measure of our problem solving ability, of our technical knowledge.

This was the very reason behind the badges, the flair, the gamification.

Even when we gave advice in comments and sometimes answered as CW or in the chat, the meaning of those points was never lost.

Those points weren't the reward for building some SEO tailored content or for making more people look at ads; they were a recognition of our abilities. This was not the whole of it; most of us big answerers are also part of many communities and FOSS projects because we're helpers at heart, but it was a core motivation.

And now, the rule changes. Now reputation seems to be a measure of the traffic we build.

This is why I feel betrayed. Because it's a contract change, it's a depreciation of what made us build SO.

37
  • 369
    The change is bad. Pulling it out of a blackbox, without consulting the user base is worse. But recalculating reputation retroactively is a huge break of trust. A falsification of history. Deceit. You can tell I am not happy with this. Nov 13, 2019 at 21:02
  • 25
    I could not agree more, great answer. Also great point @ErwinBrandstetter. Nov 13, 2019 at 21:15
  • 41
    I'm not sure reputation was ever really a good measure of your technical expertise. It can certainly be a part of it, but, mostly, reputation is a measure of activity on the site. Anything at all approaching the standards would, eventually, result in gaining rep. After you're past the initial threat of the question/answer ban, it's just a function of time spent, more or less.
    – fbueckert
    Nov 13, 2019 at 21:18
  • 37
    The harvested reputation points are more a measure of participation than anything else. On the question side, some users have literally asked 1000s of very low quality questions (you know who you are) - essentially pollution. A few questions are downvoted, but the net result is tens of thousands of reputation points. Nov 13, 2019 at 21:23
  • 10
    @ErwinBrandstetter While I disagree with the change as well, making a change and not re-calculating reputation seems unsound to me.
    – Bergi
    Nov 13, 2019 at 21:42
  • 18
    @Bergi: Think of bank accounts. If a company decided to re-shuffle wages (without consulting the staff) that would be an outrage (and typically not legal). But if they also recalculated everybody's bank account on this flimsy base, that would just destroy trust, basically deconstructing the money system. If wages change (based on a more reasonable process), past wages are not retroactively recalculated. Nov 13, 2019 at 21:52
  • 39
    Despite the popularity of this view, reputation was never any sort of measure of one's technical knowledge or problem-solving abilities. Aside from the fact that upvotes on questions have always been rewarded with reputation (albeit at a 1:2 ratio compared to upvotes on answers), even upvotes on answers don't necessarily indicate technical knowledge. Certainly not when you realize that downvotes don't even come close to canceling out upvotes. All reputation is and has ever been is a measure of your participation on this site, and thus familiarity with its rules, norms, and customs. Nov 13, 2019 at 22:05
  • 84
    Claiming that reputation would purely be a measure of participation - like @Cody just did - doesn't make sense to me. If that was the case, we users wouldn't choose whom to award points, it would be awarded, well, for participation. That's not the case. Nov 13, 2019 at 22:23
  • 103
    @CodyGray - I think most regular users would disagree with that first sentence. One could be highly active and contribute a lot to the site, but if it's all crap and nonsense, reputation would reflect that. Reputation is a measure for useful participation, and to be useful on a technical Q&A site, one surely needs some technical skill to be able to ask and answer questions, and as can be seen directly from the list of users with the highest rep, the more skill, the more reputation. You'd think technical knowledge, and lack thereof, would directly correlate to reputation on a site like this
    – adeneo
    Nov 13, 2019 at 22:25
  • 23
    @fbueckert I've always thought of reputation points as a proxy for how helpful I've been to others. But answers are much more helpful than questions, so it made sense that answers would be awarded more points than questions. Nov 13, 2019 at 22:40
  • 14
    @IncreasinglyIdiotic Not "smarter". Judging by the tooltip of the vote button, they just provide more helpful answers. (As in both "more answers" and "better answers").
    – Bergi
    Nov 13, 2019 at 23:24
  • 27
    @IncreasinglyIdiotic - I'm willing to help out as a surgeon, doesn't mean I should though, seeing as I know nothing about surgery. Providing correct answers is what gives this site value. Providing a whole lot of wrong answers, even though it is contributing, doesn't really add anyhing, other than confusion. I don't think high-rep users feel that they know more because they have a higher rep, maybe some, but certainly not most, but high-rep users are probably more likely to comment and correct wrong answers.
    – adeneo
    Nov 13, 2019 at 23:49
  • 110
    @CodyGray This has been on the reputation help page for as long as I can remember: Reputation is a rough measurement of how much the community trusts you; it is earned by convincing your peers that you know what you’re talking about. One would think that problem solving and technical abilities would be a prerequisite for convincing peers that I know what I'm talking about. I'm of mixed opinion on this change, but to pretend like reputation has always been about participation alone is rewriting history to defend unpopular changes. Nov 14, 2019 at 3:25
  • 20
    @CodeCaster I mainly get to have to hide my rep from recruiters and employers now that it's a measure of time spent, I guess. Nov 14, 2019 at 7:46
  • 9
    I get way more upvotes on easy answers to stupid questions than on detailed answers to hard questions. The more work I need to put into an answer, the more knowledge it displays, the less reputation I will gain from it. Thus, rep does not display knowledge. It displays how quickly you answer a new, easy question. Nov 14, 2019 at 14:28
491

To that end, if you have questions, concerns, comments, or other feedback about this change, please post it as an answer here.

Feedback???

To what end? Decision has already been made, regardless of the negative feedback on the Meta site. One of the questions posted before official announcements and that is not deleted is Should the weight of question upvotes be increased network-wide?

This decision will be detrimental to the quality of Stack Overflow. I believe it is final nail in the coffin.

RIP Stack Overflow, welcome Homework Overflow.


I will repeat what I said in answer to the linked question because some things you can never say enough times:

Should the weight of question upvotes be increased network-wide?

No.

  • Answers are more valuable than questions because they require knowledge, giving more reputation to answers rewards learning and acquiring that knowledge.

  • Users asking a question (trying to solve problem) will get answers besides the reputation, a solution to their problem which is a reward on its own.

  • Reputation unlocks moderation powers - reputation gained through knowledge means moderation will be done by more knowledgeable people and that reduces potential errors.

  • We don't need to add more incentive for people to ask questions, especially not on Stack Overflow, there are already way too many poorly researched and poorly asked questions.

  • Questions often get upvotes because of "me, too" problems. Someone has a problem, finds the answer on existing question and upvotes both even though question might be less than stellar or even rather poor.

As the end result, giving same reputation for questions as for answers will only reduce overall quality.

22
  • 3
    Wow, I didn't know about the Meta post. Is this the "less-than-ethical member" the OP here was talking about? Is it someone else and this is just coincidence?
    – Unihedron
    Nov 13, 2019 at 22:16
  • 2
    @Unihedron No. This is not question asked by "less-than-ethical member". There were other questions before this one but they have been deleted. I think some are mentioned in the comments, but you need 10K on Meta in order to see them.
    – Dalija Prasnikar Mod
    Nov 13, 2019 at 22:18
  • 27
    Well, yeah, feedback. I'm not running the show here, but I still fundamentally believe that the community has a voice, and as long as it's possible, I'm going to see to it that they have a reasonable way and place to express their opinions. Unilateral decisions by staff have been a fixture of this place since the Atwood days, including the original change to halve rep gains from question upvotes (announced and [status-completed] on the same day). Nov 13, 2019 at 23:08
  • 152
    @CodyGray Back in the Atwood days, those unilateral decisions were actually made for the good of the site's quality, and hence we were happy for our benevolent overlords to make said decisions. Now we have decisions being imposed for "niceness", and today's overlords wonder why they are not trusted.
    – Ian Kemp
    Nov 14, 2019 at 8:01
  • 52
    "Reputation unlocks moderation powers - reputation gained through knowledge means moderation will be done by more knowledgeable people and that reduces potential errors." This by itself is enough to convince me this is a bad move.
    – JeremyP
    Nov 14, 2019 at 9:38
  • 8
    While I agree with a lot of the sentiment in this post, I've always taken issue with the last bullet point style of thinking. Up voting on 'me too' questions. While it is somewhat possible to objectively decided if a question is good or not, really that comes from consensus. If someone asks a question and gets and answer, and others find that answer by searching for similarly phrased questions, surely that makes a question good. I get that we want 'good' questions by SO standards and to encourage that to new users, but if a question helps you find a good answer, then the question is good.
    – MikeS159
    Nov 14, 2019 at 13:19
  • 24
    @CodyGray - "I still fundamentally believe that the community has a voice" What does "has a voice" mean? How is it different from not having a voice? Nov 14, 2019 at 13:48
  • 12
    @MikeS159 Question is do the "How to format string in Java" type of questions deserve as much reputation as they have gained. Yes, you can more easily find answer to such questions here than in official documentation and answers on SO are often more detailed and better than documentation. But you reward answers to such questions, not the no-effort two line question.
    – Dalija Prasnikar Mod
    Nov 14, 2019 at 14:06
  • 15
    Reputation unlocks moderation powers - reputation gained through knowledge means moderation will be done by more knowledgeable people and that reduces potential errors. There's already enough bad reviewers in each of the queues, and the people who just blindly upvote every single question (I'm talking people with 10K+ upvotes and no downvotes) are just going to accelerate that influx now that questions are worth twice as much. Asking crappy questions and getting upvotes is easy. I for one welcome our new participation-trophy overlords. Nov 14, 2019 at 14:37
  • 11
    @Scott Umm, pretty sure the answer to that question is self-evident. The difference would look something like shutting down the Meta sites. Please don't take this as any sort of hint that is going to happen. I created this question with the express purpose of allowing community members to constructively make their points of view known, pretty much exactly as it has happened. I don't know that the feedback will have any effect, but I do know that staff members are reading it, and regardless, I think being able to make one's opposition view known has value. Nov 14, 2019 at 17:42
  • 19
    @CodyGray Please don't take "feedback" comments personally (I hope you didn't) we all know or at least hope so that there are some staff members that do listen to the community. It is not easy to express disappointments when on the other side is someone that is not even closely responsible. I am sorry that we are shooting the messengers in this process.
    – Dalija Prasnikar Mod
    Nov 14, 2019 at 17:47
  • 2
    I only wish i could upvote this twice. I think every single point you make here is the absolute truth, including the fact that they don't care at all about the community feedback on any change. I too think this may indeed by the final nail in the coffin.
    – Isac
    Nov 14, 2019 at 18:01
  • 11
    @CodyGray - one evident meaning of "having a voice" is that you can speak. Presumably you didn't mean that, because you wouldn't need to state that you 'fundamentally believe' something so obvious. Another meaning is that what we say counts for something. But I don't think you meant that. So what does 'having a voice' mean? It's okay to admit if no one cares what we think. That's generally considered not having a voice. They probably should shut the meta site down. It would be more honest. Nov 14, 2019 at 20:02
  • 9
    It counts for something to me, @Scott, and I know it counts for something to at least some of the staff members. Does it count for something with the actual decision-makers? I don’t know. Maybe? But I get that it feels more like “maybe not”, and I get (oh, so I ever empathize!) that is very frustrating. But that’s no reason to stop speaking out, both when you’re happy and when you’re not. Importantly, it should count for something to you (where “you” is all community members). Nov 14, 2019 at 20:05
  • 6
    At the end the decision makers must understand, earlier or later, that an answer is more worth than a question. And they must realize that an expert who answers someone else's question has, up to some point, the natural right to put some requirements on the standards of the question and the way it is asked. That has been the case since the early days of mankind and actually most people who ask a question will understand that. Sometimes it's just that they don't know the standard, which SO has to better communicate (since the early days)
    – hek2mgl
    Nov 14, 2019 at 22:34
301

People asking questions are already getting the additional reward of having their questions answered. Increasing reputation gain from upvotes will (in combination with the bad practice of some people to upvote downvoted questions to "balance out" the vote) only encourage people to post more questions, period.

I do not expect this change to have any net positive effect at all. Just an increased influx of bad questions.

8
  • 43
    I don't expect the change to make any difference whatsoever in the number or quality of questions asked. Nov 13, 2019 at 22:43
  • 32
    That would be the best-case scenario. Nov 13, 2019 at 23:44
  • @MarkRansom correct, not in the number or quality, but in the way they are valued
    – avalancha
    Nov 14, 2019 at 9:09
  • 6
    Bad questions shouldn't be upvoted, so why would it make a difference?
    – user253751
    Nov 14, 2019 at 10:11
  • 59
    @user253751 Because bad questions are being upvoted all the time. Welcome to reality. Nov 14, 2019 at 10:32
  • 3
    @AnsgarWiechers Then we need to address that problem. The alternative is to remove voting from questions altogether, not to leave it there but divorce it from the reputation system.
    – IMSoP
    Nov 14, 2019 at 13:25
  • 2
    "only encourage people to post more questions, period." It would appear this is the goal of the new CEO. Improve this
    – winwiz1
    Nov 18, 2019 at 22:50
  • 2
    @user253751 They shouldn't be, but they are. Very, very often. For all sorts of reasons. Nov 19, 2019 at 22:11
236

Why this?

Why now?

Why is there more value being placed on questions - which are the more inexhaustible resource in the Q&A ecosystem?

What does this accomplish beyond warm and fuzzies for the question askers who ask a reasonable question?

19
  • 60
    Must... Implement... Bounties... For... Meta...
    – S.S. Anne
    Nov 13, 2019 at 21:06
  • 4
    Why not now? And if it achieves the latter, is that not enough? Not that I have any real answers for you, but you're making it sound kinda good to me.
    – Bart
    Nov 13, 2019 at 21:10
  • 6
    @Bart: I would have valued a responsive profile page higher than reevaluating whether or not we should give questions 10 rep as opposed to 5. The reason is simple: questions are what get people to the site. Answers are what keep people on the site. I would've thought that this was a settled matter by now.
    – Makoto
    Nov 13, 2019 at 21:12
  • 10
    You know, I was on board with that for the longest time. But given the waves of utter drivel I don't even care to answer, I'm kinda okay with rewarding those who still manage to stand out. I don't really mind this change. And as for "features" that have priority, in any decently large organisation there are multiple things in development at all times. Something being released earlier doesn't necessarily imply higher priority.
    – Bart
    Nov 13, 2019 at 21:15
  • 24
    @Bart: I just completed two and a half days of Agile training which is why my mind is still pretty amped up on the whole priority thing. What I remember is, if a feature which wasn't a priority was released before other prioritized features, then that means the team isn't building the right thing. I won't speak to Stack Exchange's priorities since I'm not privy to them. But, one half of me questions who their customers are, and the other half already knows.
    – Makoto
    Nov 13, 2019 at 21:20
  • 1
    @Makoto Agile training. God that brings back memories. But I've also been on the end where gamers complained loudly about "ridiculous updates" we made while not fixing core issues, when in fact we were working hard at both, but the released updates were just easy low hanging fruit we could get out quickly. Aaaanywayyy, I'm going off topic ;)
    – Bart
    Nov 13, 2019 at 21:23
  • 8
    If you haven’t already, please read the updates to this answer, specifically the bottom non-quoted portion. I linked to it from the question, but it’s important because I think it answers what you’re asking. It explains why this—it was a mistake all along; there are better, more effective solutions to the actual problem; and it’s finally being corrected. It’s really not a warm fuzzies issue. I thought the same thing when I first heard it, until I read my own explanation. (Wait, what?!) Why not now? It takes a lot less development work than building a new profile page. Nov 13, 2019 at 21:36
  • 30
    @CodyGray: I skimmed that answer and - in fairness - it was trying to justify this. I don't agree with the justification, because it's basically stating that there were no pearls in answers, and that both questions and answers are equally valuable. This is categorically false. While a good question can spark a great answer, the world only really cares about the great answers, empirically demonstrated by me not answering any questions for the last, say, six months, and only looking for answers to questions (and sometimes failing at that).
    – Makoto
    Nov 13, 2019 at 21:43
  • 18
    I'm growing tired of shouting to the wind. Triviality does not factor in here. I'm talking about value. Value to the customers. If it's the case that Stack Exchange has decided that their definition of "customer" now means "users who just ask questions", then I am wasting my time. If it doesn't, then we get to look at others - like myself - who use the site on a mobile device.
    – Makoto
    Nov 13, 2019 at 21:56
  • 7
    A cynical person might say that the intention is to punish the people who have answered many more questions than they have asked and then participated in MSE/MSO to downvote some recent changes. The punishment is that their relative reputation is reduced, e.g., overnight I have gone from page 220 down to page 251 in the user reputation leagues. I have been devalued by more than 10%. But it could plausibly be construed as an unintentional side-effect. Nov 14, 2019 at 9:36
  • 9
    "What does this accomplish beyond warm and fuzzies for the question askers who ask a reasonable question?" Nail meet head Nov 14, 2019 at 10:37
  • 2
    Do you guys really think that it is us regular (and not so much) users who are the customers for the site? We are equally as much customers for SO as we are for, say, FB. Job advertisers are the real customers here and they demand more conversion in exchange for money. And when money start to play their part everyone else shuts up. All that recent welcomeness bs is only an excuse for the owners' greed for more $.
    – ayorgo
    Nov 14, 2019 at 12:07
  • 1
    @ayorgo: It's not like the question askers are bringing money in, either.
    – Makoto
    Nov 14, 2019 at 18:22
  • 2
    @Makoto they increase chances of ads being clicked on I guess. And since the no-of-answerers to no-of-askers ratio is pretty small, someone high up the management ladder with a narrow understanding of how the site works may be pushing this to increase the total audience at the expense of quality to make the quarterly report look better or smth. At least this is the only way I can make sense of what's going on.
    – ayorgo
    Nov 14, 2019 at 18:37
  • 6
    "What does this accomplish beyond warm and fuzzies for the question askers who ask a reasonable question?" Warm and fuzzies for people who ask unreasonable questions. Also warm and fuzzies and a sense of self righteousness for the people demanding it.
    – jpmc26
    Nov 14, 2019 at 21:42
177

The main problem, for me, is that asking questions doesn't require you to become truly familiar with the site and its quality standards. And handing out privileges that require familiarity and knowledge of the site is problematic.

Answering a substantial amount of questions requires users to show an interest in the site and shows a willingness to help others. Those are the users I'd like to have reviewing questions, and would trust with the privileges that come with reputation. Asking a substantial amount of questions, however, requires only visiting the site when you have a question (instead of actively visiting to look for new questions), and can be motivated purely by a selfish need. For these users I have strong doubts.

Take the first edit of this recent HNQ for example. That would've netted this user 160 rep already, and assuming this bug hangs around, might very well get in the neighborhood of 2K over a couple of months. This same user has edited in Help!!!!!!!!!! after Please Help was removed. I'd rather not have such users gain access to review queues or the privilege to edit without going through the queue, even though their contribution might be valuable.

9
  • 9
    You’d rather that user gain only 80 rep, and only ~1k over a couple of months? Aside from that, I’m not convinced that posting answers really shows that one groks the purpose of the site, either. Nov 13, 2019 at 21:01
  • 114
    @CodyGray - I'm not convinced the people running the site groks the purpose of the site either. Most of us do this solely for fun and personal learning, and the fake internet points are fun and competitive, nothing more, they don't pay the rent, yet we care enough and have worked hard enough, that just giving them away retroactively seems wrong. This all worked 8 years ago, users joined and become active members back then as well, and if it isn't working now, something has clearly changed, and it probably isn't the community?
    – adeneo
    Nov 13, 2019 at 21:05
  • @adeneo StackOverflow isn't " just giving them away". They are awarding participation points equally to the two groups of people participating with the site. Nov 13, 2019 at 23:22
  • 37
    "Aside from that, I’m not convinced that posting answers really shows that one groks the purpose of the site, either." wait, WHAT? Nov 14, 2019 at 4:07
  • 2
    The problem is probably not the reputation count itself (though I'd prefer a seperate count for Q+A reputation), but the fact that privileges are based on reputation. That's why it seems unfair. IMO privileges shouldn't be awarded based on reputation but rather earned based on participation and other factors. Perhaps earning badges should unlock privileges, giving more value to badges and at the same time providing the evidence of participation.
    – user4864425
    Nov 14, 2019 at 11:33
  • 1
    If votes on questions are meaningful at all, then asking questions that receive positive votes does require the same familiarity with the site as writing answers that receive positive votes. If votes on questions are not meaningful, then that is a problem with the voting system, not the reputation system.
    – IMSoP
    Nov 14, 2019 at 13:27
  • 2
    I think there should be a system in place where a newbie has to answer 5 questions and get at least one (1) upvote on them all before gaining the privilege to ask questions. This way there would be more incentive for newcomers to compose high quality questions already from the beginning, contributing to the community from day one. One could even give the first 5 upvoted answers a x2 reputation bonus just to make it even more attractive and rewarding for people who just joined. But after that go back to how it always has been.
    – silkfire
    Nov 14, 2019 at 16:03
  • @silkfire ...and if it was in place, I would probably never use SE at all. I started on ruSO with Lua questions, which is not nearly as popular as JS, not to mention that ruSO is far less active than main site. If there is one good Lua question per day on ruSO, it is a very good day. Nov 21, 2019 at 9:12
  • Imho reputation shouldn't earn privileges at all. It should be just for bragging rights only. It's the fallacy of the "Peter principle".
    – Calmarius
    Nov 21, 2019 at 16:50
158

Yay for better rewards on good questions, but where are the rewards (or acknowledgement) for finding duplicates?

I can imagine this change will create a brief rise in voting rings and duplicate questions asked by users only interested in reputation with no regard to overall quality so to me, it will be more beneficial for the "Stack Overflow veterans" by rewarding them with their experience in locating existing questions. There aren't many >20k users asking questions regularly since they're able to use the 12 years of existing content effectively.

11
  • 15
    I like this. Finding accurate dupes is so terribly under-rated. But, it's "Stack Overflow" not "StackOverflow" ;-) Nov 13, 2019 at 20:48
  • 3
    What, isn't fighting other gold badge holders who reopen the question you just dupe-voted, because they had just typed an answer when you did so, reward enough? (And no, that's a bit tongue-in-cheek, this doesn't happen more often than me wrongfully dupe-voting).
    – CodeCaster
    Nov 13, 2019 at 20:49
  • 3
    @CodeCaster - Its not a real fight unless they're over 100k rep and telling you you're making it an unwelcoming place too
    – Sayse
    Nov 13, 2019 at 20:55
  • The r-word has been banned (and removed from The Many Memes of Meta). Help vampire is also on its way out. Nov 13, 2019 at 21:39
  • @PeterMortensen - Interesting, I didn't know that thanks. I'll have to start using something else to call these users instead but nothing else I can think of at the minute has the same ring to it. Note: I'd never call a user this directly, I refer only to the act of "rep whoring" and its participants as a collective
    – Sayse
    Nov 13, 2019 at 21:51
  • 27
    I'd upvote this 100 if I could. I spend most of my time in the PHP tag, and most of my time is spent finding dupe targets. The questions that are actually worthy of new answers are few and far between.
    – Patrick Q
    Nov 13, 2019 at 22:03
  • 12
    I don't care that much about getting more reputation from closing questions. Instead we should devalue reputation gained from closed questions, as has been proposed so often.
    – Bergi
    Nov 13, 2019 at 22:16
  • I don't think that now reputation means what it used to. This change is similar to printing money, you have more, but they become worthless... Nov 14, 2019 at 9:24
  • @meJustAndrew That would only be true if the amount you need to have to get the privileges also increased proportionally Nov 14, 2019 at 12:42
  • I don't say it lost its value from a functional point of view, as usage on the site (even though I believe that should be done as well, if we will keep on having this change) but from a psychological point of view. When I was looking to a higher rep user, I was having some sort of intuition of his contribution. Now that changed a bit. Nov 14, 2019 at 12:57
  • "but where are the rewards (or acknowledgement) for finding duplicates" I don't know if we should get rewards for these, however, I also think that people shouldn't get rewarded for answering questions that are duplicates. I constantly see a couple of users that consistently answer obvious duplicates when they should be closing them (as they're a "gold badger").
    – Thom A
    Nov 14, 2019 at 17:07
149

As a longtime Stack Overflow contributor who has answered over 2,000+ questions and asked none, it feels like this is inadvertently affecting those of us who have primarily answered questions as opposed to asking them.

What exactly is the benefit in doing this retroactively? Why rewrite reputation?

Aside from that, this change is still disproportionate and skews reputation because question downvotes still only result in a net loss of -2 reputation.

At least double question downvotes to -4 reputation to balance out downvotes against upvotes.

25
  • 15
    I’m not clear on how this disadvantages you as an answerer, rather than an asker. By definition, it has no effect on your reputation earned. This is not a zero-sum game. It seems perfectly valid to have two different routes to earning privileges: asking or answering. The benefit of doing this retroactively is fairness. Also see my retrospective linked from the question: essentially, Jeff’s original decision to reduce rep upvote weight has been deemed to have been a misstep, so it’s being reversed with the benefit of hindsight. The upvote/downvote disproportionality is there for answers, too. Nov 13, 2019 at 21:13
  • 79
    @CodyGray - That is exactly it, my reputation remains unchanged along with anyone else who has never asked a question, yet we are retroactively increasing everyone else's reputation which consequently devalues the reputation of those who only answered questions. It's basically conditional reputation inflation. Nov 13, 2019 at 21:21
  • 21
    I agree, I've answered 7500'ish questions, and never asked one either, so this change doesn't affect me at all really. Still, a lot of other users will quite suddenly receive a humongous amount of reputation from the retroactive change, users ranging from many of the sites top users, that have created canonical questions that they have answered themselves, to be able to close questions on common topics, that have thousands of upvotes over the years, to users that haven't visited the site in years, but asked a really basic question 13 or so years ago.
    – adeneo
    Nov 13, 2019 at 21:45
  • 7
    The relative ranking changes will be interesting when they get recalculated in a couple hours. Except for the very top scorers, I expect question answerers will drop, and askers will zoom up. Nov 13, 2019 at 22:57
  • 17
    @CodyGray Why asking questions should mean earning privileges? Let's say I decide to spend an hour answering questions. It will be hard work, I will have to be quick, have the knowledge, often do some research, etc. I can now spend that same hour asking questions, I can ask many more questions than I can answer. In extreme scenarios, I could just figuratively throw shit on the wall hoping that some of it will stick, and if it will, I will eventually get moderating privileges. This makes very little sense. Again, it seems like VIEWS > MERIT, as this earns SO money.
    – Varin
    Nov 14, 2019 at 14:42
  • 15
    @CodyGray it does change a lot for the answerers vs the askers. It changes their ranking. This retroactive goalpost shifting has cost me 255 places in my ranking. The rep number in points is meaningless in itself, it's the ranking that counts. Just like with money, there's inflation with time, so points have no meaning except through the derived ranking. And you have just changed that meaning. ReTrOaCtiVeLy . Make whatever friggin change you like, but doing it ReTrOaCtiVeLy is beyond outrageous.
    – Will Ness
    Nov 14, 2019 at 15:44
  • 13
    The fact that this incentivizes asking questions is what worries me. Yes, we want people to ask questions, but I'm still of the opinion that asking a question should be the last thing you do on your research road. I wonder if this was a reason that answer was deleted at one point.
    – zero298
    Nov 14, 2019 at 15:58
  • 1
    @WillNess No one looks at that for any reason other than to look at it though. It has no bearing on anything other than itself. Aside from that, your ranking can change anyway by that many positions (or more) just from normal site activity.
    – TylerH
    Nov 14, 2019 at 16:03
  • 3
    @zero298 I'm sure it was part of it. The company (including the CMs) care about profit because it's what benefits them as a company and as individuals. The bottom line is quality harms profit, so they're doing what they can to eliminate it.
    – TylerH
    Nov 14, 2019 at 16:05
  • 1
    @TylerH of course it can change, and does / did, according to the rules. And now they changed those rules, retroactively. that's the only thing I'm objecting to, really; otherwise I don't care what they do with their site. By they changed what I did here, in effect.
    – Will Ness
    Nov 14, 2019 at 16:19
  • 1
    @TylerH also it does have a bearing on one more thing: that "top 0.5% overall" next to a user's name. So yes, people do see that, a lot.
    – Will Ness
    Nov 14, 2019 at 16:21
  • 5
    @WillNess OK, let me rephrase: no one cares for any reason that matters. If someone is changing how they interact with you in real life (or even here on the site), either by offering you some respect, a job offer, a book deal, a conference gig, etc. vs not doing those things, based purely on whether you're top 0.22% vs top 0.20% on Stack Overflow, then you're better off not dealing with them at all in the first place. They're clearly placing way too much value in something that doesn't accurately represent said value and never claimed to in the first place.
    – TylerH
    Nov 14, 2019 at 16:40
  • 7
    Feels to me like SO is saying "We don't really care about you helping people and sharing your knowledge - We value more people asking for help", which doesn't resonate well with me. Nov 14, 2019 at 20:43
  • 3
    @TylerH yeah, literally no one cares. let's eliminate the rep entirely then, no one cares / should care about it in RL either.
    – Will Ness
    Nov 15, 2019 at 8:53
  • 2
    @Steve-o169 You can identify a fairly new tech, identify questions that have lots of upvotes in an older, similar tech and ask literally the same questions under different tags. If it's only 6 questions per day, that will take me 5-10 minutes with some prior investigation. The main point is: questions != answers in terms of how valuable they are, therefore they should not carry same weight. I gained 20% extra rep through this change and I rather give it back as I don't feel like I earned it.
    – Varin
    Nov 18, 2019 at 15:28
99

I'm really not sure that this is going to really help things, if I'm honest. Something that many of us see (especially at the weekends), are a lot of low quality questions. A lot of new users don't really bother to take the time to read the tour, or the help pages, and especially understand what an MRE is, making their questions difficult to answer.

From someone who solely contributions against , and its version tags (e.g. ) one of the biggest problems I see on an hourly basis (possibly more) is a question showing no SQL, and images for data, and no effort. These questions are truly awful from a SQL point of view. The data isn't consumable so if I (or any one else) wants to contribute and test our answers we have to transcribe that information, which can be very time consuming. But these questions do get upvotes from people, as it "looks" like the OP has supplied the information we "need" (it's just useless to anyone who wants to help). This means that it now takes 5 down votes to negate that, not the 2.5 it used to.

I'm actually for new users getting the needed reputation to be able to contribute (being able to comment on other questions is a big part of being able to contribute on any SE site), but I think there needs to be a better way than this. Perhaps, rather than doubling the reputation you get for an upvote on a question, you should get more on accepting an answer. Acceptance rate can be pretty low, and for a new user I think getting them to understand the importance of it, with a bigger reputation "carrot" could be a good way.

Hey, perhaps a question that gets a lot of high voted (define high) answers should reward the asker too. If an answer to the question gets 10 votes, the asker gets 10 rep; and repeat for each answer (and each 10 of votes, i.e. @ 20, 30, etc). There are some questions on the SE site that have 10's of answers and many of those can have 50+ votes. That would be a really good way to rewarding the asker, in my opinion. They have asked a good question and got upvoted for that, and also they attracted some great answers, which has contributed to the community; and the asker is (effectively) rewarded for good answers as they were the reason they were contributed to the site.

Edit: Something I've noticed very quickly from myself is that since this announcement I am now far quicker at downvoting questions. This is something that previously I wasn't; I would comment first, wait for a response, and downvote accordingly (if no response received or request for improvements were ignored). That isn't true any more, I'm downvoting with comments (or upvoting an existing comment). This is especially true for the question types outlined above (lack of MRE, images for date, etc, like this, with no code to even debug but has an upvote).

Why am I now quicker to downvote? Because these truly poor questions should not be getting 10 reputation from a a single vote, when there is not a single sign of effort, or useable piece of code/data. I very much doubt that my changed behaviour is an intention of the outlined changes in the OP.

10
  • 28
    Nice ideas here, all better than the current change.
    – Unihedron
    Nov 13, 2019 at 22:03
  • 8
    Up you go! The one with acceptance of the answers should do it, especially because many newcomers do not bother to accept an answer Nov 13, 2019 at 22:05
  • It sounds like the problem here is "these questions do get upvotes from people". That needs fixing regardless of what is done to reputation, because scores are also used for other purposes.
    – IMSoP
    Nov 14, 2019 at 13:31
  • Fully agree. moreover, I will now actively go through my old upvotes on questions an will unupvote any that I find I did as an encouragement, on the more poorly asked questions. Yes I did that a lot. Now I won't. An I will downvote without hesitation now, too.
    – Will Ness
    Nov 14, 2019 at 15:52
  • You probably won't be able to unupvote, @WillNess ; not unsure the question has been edited. Unless they don't have the same rules as answer upvotes.
    – Thom A
    Nov 14, 2019 at 16:05
  • @Larnu oh I'm editing them, be sure of that. Any that I find lacking, and not deserving of my upvote under the new value system, I'm editing and un-upvoting. The good ones I don't touch of course.
    – Will Ness
    Nov 14, 2019 at 16:11
  • Editting to unvote seems like an unexpected loophole. :/
    – Thom A
    Nov 14, 2019 at 16:16
  • 4
    heh. they could've discussed it with. the. community. first, could they? :)
    – Will Ness
    Nov 14, 2019 at 16:39
  • 7
    I had the same thoughts about being more likely to downvote if question downvote rep isn't also increased. I can easily justify a "lack of research effort" downvote on a lot of questions. Nov 14, 2019 at 23:18
  • 1
    Some really good points as alternatives with reasoning +1. Especially the answer upvotes to asked question reps which for sure improves and encourages the site quality content wise. Nov 17, 2019 at 8:09
91

I will be honest, I mostly down vote. I exhaust my votes almost every day and 98% of it are down votes. It is much more difficult to find either a good answer or a good question to up vote. On one hand this change makes me happy, because people who asked really good questions will be rewarded, but this has also a very serious adverse effect. 2 reasons:

  1. Sometimes I make exceptions to my voting habits and I upvote the question purely because the answers are extraordinary and I want this question to pop up in the search more often. This is bad... I feel bad doing this, because I reward a bad question for wrong reasons. But I know that if someone searches for a duplicate target they do not see the answer score, they see the question score!

  2. We edit the questions! People post awful questions. Sometimes they are completely unreadable. We as a community edit them to make them at least answerable and if they attract good answers, we tend to polish those questions even more. The end result is that the extremely poor effort of the asker might be rewarded due to our hard work in editing.
    We are even encouraged to edit the questions when we answer. I think there are few badges around this topic. Why is the OP, who couldn't care less about the quality when they asked it, be rewarded for our hard work?

10
  • 19
    "I will be honest, I mostly down vote. I exhaust my votes almost every day and 98% of it are down votes." I'm not sure that's something to be "proud" of (I'm not saying you are). Using all your votes in a day mainly on downvotes is a significant number. I admit, I'm sparing in my upvotes as the difference in rep is a lot, but I prefer to comment and explain first. If I get no response, or my (polite) request for improvements is batted away, then I downvote. but I don't think I'd ever hope to go through a day and use "98%" of my daily votes on downvotes.
    – Thom A
    Nov 13, 2019 at 22:12
  • 44
    @Larnu I invite you to web and php tags. Try to find a decent question to answer.
    – Dharman Mod
    Nov 13, 2019 at 22:14
  • I don't doubt it's as bad as the sql tags, don't worry @Dharman . :) (I see a few of the bad php ones from the SQL side anyway. Loved one today which said that the results were different in PHP to SSMS, but provided no code, and no data /facepalm .
    – Thom A
    Nov 13, 2019 at 22:15
  • 9
    When I was actively voting, I tended to be very reserved about downvoting questions. If I were still active, this would be the go-ahead to hand out question downvotes without even thinking twice on all but actually good questions. Nov 13, 2019 at 23:03
  • @hatchet-ReinstateMonica Fully agree. Even more, I will now actively go through my old upvotes on questions an will unupvote any that I find I did as an encouragement, on the more poorly asked questions. Yes I did that a lot. Now I won't.
    – Will Ness
    Nov 14, 2019 at 15:55
  • 3
    @WillNess Then it sounds like you were voting wrong, and breaking this habit is a good thing. For votes to be useful for anything, whether it's reputation or just ranking search results, they need to reflect quality, not sympathy.
    – IMSoP
    Nov 15, 2019 at 13:10
  • @IMSoP got the message: "no more being nice and welcoming". as I said somewhere else on this entry, I only did this for Qs which were good at the core and only superficially - or initially - bad, which if I felt received their downvotes unfairly. I'll be more skimpy with the upvotes now. that's just how it is.
    – Will Ness
    Nov 15, 2019 at 16:15
  • 3
    @WillNess That's not what I said at all. Feel free to be nice and welcoming but don't use the up vote button to do so. Maybe leave a note saying "there's the beginning of a good question here, but you need to fix these things before it deservers an upvote".
    – IMSoP
    Nov 15, 2019 at 16:53
  • 1
    The community also edits answers. Are there any reputation-sharing mechanisms for answers that don't also apply to questions?
    – alltom
    Nov 16, 2019 at 4:43
  • 2
    Downvoting is important. don't feel guilty using it. It's the only way to sieve the sand. Don't feel guilty about upvoting questions due to them bringing in good answers either because that meams the question was useful.
    – zero298
    Dec 2, 2019 at 13:37
89

I believe the approach of this is entirely wrong.

There are for sure positive and negative effects of this change which can be debated for long, I am not condemning the change itself, but the way it has been taken:

Unfortunately, a less-than-ethical member of the larger Stack Exchange moderator community leaked this private communication to the public...

I believe that the right approach here would have been to let the community decide if this change should be done, or not, as it changes the community and the interactions between the people here, not the site, the ads, or things more related to the site.

This attitude is the thing that should really be changed!

12
  • Up'd for "This attitude is the thing that should really be changed!".
    – Unihedron
    Nov 13, 2019 at 22:13
  • 14
    I agree that the community should have been consulted, and that they should have had a chance to raise their objections first. Asking the moderators was a good and very welcome first step, but it isn't sufficient going forward. That said, you are incorrect in implying that The Powers That Be making unilateral decisions without consulting the community is something new. It really isn't. Jeff Atwood was the one who said: "Listen to Your Community, But Don't Let Them Tell You What to Do". Nov 13, 2019 at 22:17
  • 9
    Jeff's original decision to decrease the weight of upvotes on questions (the change this one is effectively reversing) was not reached in consultation with the community, either. It was a unilateral dictate made by Jeff. Note the date on that blog post: March 19, 2010. That's the same date on the associated Meta Q&A, and, in fact, the same date that said Meta Q&A was tagged [status-completed]. Most of the community seemed to agree, but that was a happy coincidence. Nov 13, 2019 at 22:20
  • 8
    Aside from that, the reason I make the claim about ethics that I do is because the moderators were specifically requested not to share the information in public, as a condition of their having been given priority access to the team's plans. Whether or not you think the community should have been involved, disclosing information that you are explicitly asked not to disclose is a major violation of trust, and puts moderators (as representatives of their community) in the same boat as what we've been accusing Stack Exchange employees of doing: violating trust. It's not a good look. Nov 13, 2019 at 22:22
  • 13
    Does it violate work ethics? Yes, absolutely. I wouldn't do that myself, I don't want to break trust. But when someone finds it such an important matter that they prioritize their own moral values above work ethics, try to think about where they are coming from, because calling out poor ethics doesn't surface the problem, it hides it.
    – Unihedron
    Nov 13, 2019 at 22:26
  • 7
    That might be a compelling argument if magic Internet points were somehow a moral issue, @Unihedron. We're not talking a whistleblower announcing that Stack Exchange is funding genocide in third-world countries. Furthermore, there was no brink. The discussions between staff and moderators were still ongoing. We were still having a productive exchange about our concerns and how to modify the proposed policy to maximize benefit to all parties. Anyway...this will be the last thing I say about the "leak", because it has nothing to do with the discussion at hand. Nov 13, 2019 at 22:56
  • I'm not interested in rep, I'm merely backing the "This attitude is the thing that should really be changed!" sentiment. Passive-aggressively calling out on a moderator who would otherwise be and have been representing and acting in the interests of your own community is pretty insincere, even if justified. There are other ways to go about it, like accounting for it in far more appropriate ways than rambling about it in a one-sided announcement main post. That's, of course, my own opinion based on what I've seen, I'm not here to teach you what to do (and obviously no such authority).
    – Unihedron
    Nov 14, 2019 at 12:38
  • 4
    @CodyGray I want to thank you for all the effort you are putting into communicating to everyone this change and for providing a bridge between the community and the SE team. You deserve being a moderator, and you are doing an amazing job here! Nov 14, 2019 at 15:27
  • not the retroactive aspect of it. that just can't be condoned in any way.
    – Will Ness
    Nov 14, 2019 at 15:57
  • 12
    Regarding parallels to decisions made by Jeff Atwood - Jeff had a lot more credit in the trust bank than the current SO management does.
    – BJ Myers
    Nov 15, 2019 at 16:46
  • I've watched this echo chamber effect escalate over the last 2-3 years now driven by the face of SE apologizing or allowing for the interpretation of an admission for company decisions that won't be reversed. Now SE is in a position where most changes they make are met with hostility and frankly melodramatic reactions. SE being SE, I'm sure they're aware of this schism, but one of two things need to happen. Anticipate common reactions and address them pre-emptively when making & posting the decision, or if the motives aren't aligned with users / can't be mentioned, then don't apologize for it.
    – kettlecrab
    Nov 25, 2019 at 6:22
  • @person27 I believe you are right about the two potential ways of developing the interaction between SE and the community further on, but we will see how things evolve... Nov 25, 2019 at 9:28
87

I'm quite neutral towards this change; I won't go throwing around terms like "reputation inflation" and "welcome wagon", because I couldn't care less how much reputation other users have. What I am wondering is: who will this benefit, and how?

As far as I know, a question asker couldn't give a rat's ass whether their question is at +5 or -5, as long as they get an answer. They also don't care whether that question gains them -10 or +50 reputation. Newbies ask questions because they want answers, not because they want reputation points.

If someone wants the most basic privileges, they'll have to edit a few posts and post a good answer or three and they can vote and comment.

So again: what's the benefit, and for whom? I can't imagine a help vampire seeing a +5000 reputation recalculation along with a nice message in their profile go "Oh now I'll spend some more effort at my next question!", for example.

8
  • 68
    I don't have real strong feelings either. But at the moment it feels like, "We're getting negative attention for awful things we did. Here's some free points!" Nov 13, 2019 at 20:21
  • 5
    Many users want to comment (in general, not just related to their own questions) and now it only requires half the effort (if only asking questions). Nov 13, 2019 at 20:56
  • 6
    The data from here suggests people do care what their question was voted. Getting an upvote is on par with getting an answer when bringing someone back. Getting a downvote discourages some people from returning, even if they get an answer. And if you're in a niche tag, you can expect to get no votes in either direction.
    – Troyen
    Nov 13, 2019 at 21:13
  • 7
    @Troyen that's about votes, not reputation. An upvote is an upvote, whether it yields 0 or 10 reputation.
    – CodeCaster
    Nov 13, 2019 at 21:13
  • 2
    I was addressing your line "As far as I know, a question asker couldn't give a rat's ass whether their question is at +5 or -5, as long as they get an answer." I also contend that the amount of points earned from votes do matter. It takes me longer to research and write questions on some other SE sites than it does answers.
    – Troyen
    Nov 13, 2019 at 21:23
  • 1
    Maybe we could rephrase the question to "Why do we consider questions to be worth less than answers" Is a question that 10 people think was good or useful worth half as much as an answer that 10 people found useful? There is also the fact that people tend to only upvote the answer that helped them and not the question that prompted the answer.
    – Qwertie
    Nov 14, 2019 at 5:56
  • There are those of us out there who consider ourselves "askers" not "answerers". Sure, getting the answer itself is fantastic, and I don't really care about this change either way, but we do exist. :)
    – user736893
    Nov 14, 2019 at 19:44
  • 2
    "What's the benefit, and for whom?" As someone who was just given access to the review queues for the first time as a result of the recalc, I can't help but wonder if this isn't part of an ill-conceived strategy by Stack staff to try and make up for the recent mass exodus of moderators by throwing new privileges at everyone.
    – jmbpiano
    Nov 18, 2019 at 19:49
82

My real issue with this change is this graph (lifted from the Stack Overflow Data Analytics page, a 25k+ user privilege):

q&a graph

Since the initial drop after the site peaked in 2014, questions have actually stayed reasonably consistent—but answers have dropped to the point where there's now around the same number of answers and questions. This either means that more questions are only seeing a single answer, or more questions are simply going unanswered. (In reality, it's probably a bit of both.) I don't believe that is the model where SO works most effectively (it instead being that there's multiple answers per question generally, and the best one is voted up.)

This says to me that, as a community / site / group of users etc.—we're struggling to effectively answer most of the questions we have already. I can't see how a focus on encouraging more questions is going to help that trend.

9
  • 30
    This graph makes sense because a lot of questions have been asked before. Instead of answering them, they are closed as duplicates, where most of the time OP still gets their solution even not through an answer.
    – Unihedron
    Nov 14, 2019 at 12:45
  • 5
    I can only speak to the tags I mainly work in, more specifically ms-word: There are lots of unanswered questions, mainly because poor questions are not getting enough close/delete votes to remove them from the site. Also, there are so few people answering in the tag that it's unusual for there to be more than one reply. Side note: even when an answer is posted it's often not marked as "the" answer, commented on by the OP or even up-voted. Nov 14, 2019 at 13:58
  • 1
    I found a similar graph (by month) with better axis labels data.stackexchange.com/stackoverflow/query/59302/… Nov 14, 2019 at 14:16
  • 8
    the query doesn't filter out questions with negative votes or questions that are deleted. Also answers that are downvoted are not filtered out either. I adjusted for that data.stackexchange.com/stackoverflow/query/1148533/… as you can see there is a slightly wider gap now. The original point still stands, of course. Not enough answers compared to questions Nov 14, 2019 at 15:36
  • 1
    What is the dips? Christmas? Nov 14, 2019 at 15:47
  • @PeterMortensen Yup. Nov 14, 2019 at 15:56
  • 9
    What happened in 2014 to cause a drop of more than 20% in answers over 3 months?
    – NotThatGuy
    Nov 14, 2019 at 23:02
  • 5
    @NotThatGuy meta.stackoverflow.com/questions/320223/… Nov 15, 2019 at 9:38
  • FWIW, it's happened - as of Feb 2020 there now seem to be more new questions than new answers. Feb 22, 2020 at 0:22
67

No one is talking about it, but another issue is bounties. We need 75 rep to set a bounty which means only 8 upvotes instead of 15. The smallest amount is 50 which is also only 5 upvotes instead of 10.

Basically, it's now easier to add bounty to a question and make it featured, and it's also easier to get back the amount spent since the question is more visible, so it is more likely to get upvotes (5 upvotes are faster than 10).

In other words, featuring a question will almost cost nothing for users, so we will get more and more featured questions that are poor.

I guess it would be good to also revise the threshold of the bounty privilege and maybe double the amount of bounties from [50 - 500] to [100 - 1000].

1
  • 14
    Another valid point raised. Which also reflected by the bad quality bountied questions even before the change.
    – Mukyuu
    Nov 14, 2019 at 9:45
61

Unfortunately, I think this will only encourage help vampires.

As I've written in my reply to the blog post - I have 16 questions and almost 3000 answers. If things were reversed, should I still have the 64K reputation I have, or should it be significantly lower?

47

Upping the reputation points of questions adds to the problem of not showing the score of a question correctly.

The problem is NOT that downvotes do not count more, but how they are shown on the question/answer.

Downvotes are shown in Stack Overflow as the same as upvotes. The "Score" for a question or answer that has had two upvotes and four down votes shows as -2. But really, the "Score" of this question/answer is 1.2. (Calculated: (20 - 8)/10)

So many times, someone will see a question with a negative score and give them a "pity vote". The pity vote far out weighs the downvote. This rewards bad question asking by gaining easy reputation.

To be clear, I don't really care how much downvotes count for (though I think they should be more), but whatever they count for needs to show in the score of the question/answer.

On a more personal note, I ask way more questions than I answer. I recall years ago, when I was just about to break into the community moderator tools, the change was made to make a question upvote count 5 (instead of the 10 it was back then). It caused my rep to be cut almost in half.

I felt really betrayed. I work hard to ask really good questions (I have 100+ Stack Overflow gold badges related to good questions.) I felt that Stack Overflow was devaluing the extra work I put into good questions.

In the end I got over it because (as others have noted) I was still getting the reward of an answer to many of my questions.

I will happily accept the reputation boost, but I can really identify with those who feel betrayed because their answer-based reputation points just got devalued by half (compared to question-based reputation points).

5
  • 1
    Do you have a source for "So many times, someone will see a question with a negative score and give them a "pity vote"." ? Nov 14, 2019 at 13:30
  • 12
    @Pureferret: *raises hand*. If I see a question at -1 that should in fairness be at 0, I tend to give it an up vote, which I wouldn't have done if it already was at 0. In these cases I don't as much want to award the question, as I want to encourage others to answer it.
    – AkselA
    Nov 14, 2019 at 20:05
  • @AkselA then that's not pity voting that's doing something for the community Nov 14, 2019 at 20:30
  • 1
    I've done that.
    – onnoweb
    Nov 15, 2019 at 21:41
  • 8
    @Pureferret - I've seen it done. I've seen COMMENTS statiing it's the reason for upvotes. Heck sometimes I did it myself, though not as a policy. Both on SO and smaller sites
    – DVK
    Nov 16, 2019 at 4:09
42

...in recognition of the fact that useful, high-quality questions are just as valuable as high-quality answers.

This is factually false. A well written question with no answers is not helpful to the vast majority of users who share the same problem. Only an answer can save them time and effort finding a solution. A good question derives the vast majority of its value from the existence of answers attached to it.

This change continues SO's trend of making decisions based on incorrect premises.

8
  • 5
    Good questions do tend to get answered so the problem of good but unanswered questions is mostly hypothetical. When looking for duplicates, a garbage confusing question can make it a worse target. Especially if it means answers to the real question are cluttered with addressing other problems and side-tracks so even the answers aren't a good simple answer to a single problem. So depending on context, they can be almost as useful in the long run. Nov 14, 2019 at 23:30
  • 2
    @PeterCordes It appears that only posts bad enough to get deleted are unlikely to get an answer. Even 82.7% of closed questions have answers. So whether a question is good has little bearing on whether it's answered, unless you're going to assert that over 80% of closed questions are good. So the value in writing good questions is quite low. It's the answers that make a question valuable, and a bad question can be cleaned up later if it receives a good answer.
    – jpmc26
    Nov 17, 2019 at 1:43
  • 2
    You misunderstood my point. As you say, most questions get answered, and almost all good questions get answered. The value in a good question is not just that it has answers (almost all questions get that) but that it makes a good place for canonical answers without a bunch of confusion that side-tracks and clutters answers. And that it makes it easy for future readers to see if they have the same problem by clearly explaining what the question is. Nov 17, 2019 at 2:07
  • Agreed that questions can be simplified and cleaned up in many cases, and the editors who do that work to make a good canonical target get no rep reward, while the original author of an originally-terrible question reaps the rep reward. I don't support bumping question upvote rep all the way to 10. Maybe 6 or 7, but until there's more connection between question upvote and contribution of the author to question quality 10 doesn't make sense. Nov 17, 2019 at 2:11
  • @PeterCordes Even in your explanation, the value of a good question is contingent on the value of the answers it helps readers find. That is my point. Many well written but obscure questions go unanswered, but these are not particularly valuable. I'm not saying a well written question is worthless, but it's certainly not very valuable all by itself. As such, good questions are not "just as valuable" as good (or sometimes even poor) answers.
    – jpmc26
    Nov 17, 2019 at 2:28
  • Many well written but obscure questions go unanswered Really? That claim was missing from your answer which is what I was commenting about. If they're well-written and on topic, questions tend to get answers eventually. I agree with your argument that answers are what have real value to people searching, and should be rewarded with more rep. But not all of your reasoning to support that really holds up, in my experience. (And questions have value in helping make the site more searchable (but mostly just with useful titles which are often written by editors) Nov 17, 2019 at 2:41
  • TL:DR: I don't disagree with most of your points, just the one I commented about originally. Nov 17, 2019 at 2:43
  • @PeterCordes Many only because the volume of questions is so large, not as a percentage. (Because obscure questions are practically by definition uncommon.) I mention them because they do exist and because they demonstrate my overall point: that questions are inherently less valuable than answers, even though good ones have some value. Your mentions of good questions as a nexus for good answers further reinforces my point, that it's the answers that provide the far greater share of value. I've made an edit that may make the answer more palatable to you.
    – jpmc26
    Nov 17, 2019 at 2:44
40

This basically makes downvotes on questions even less useful. Questions now can have even more negative scores and still propel peoples reputation towards privileges that can impact the site quality. Why bothering to downvote on questions anymore? I don't think this will end well.

Good answers are even more valuable than good questions. That should be reflected in the rep gains. As it is now, it is kind of a regression and in my eyes, away from the optimum.

Time will tell how quality of questions will develop, but I did not have the impression that +5 rep for every upvote on a question was a problem that needed solving.

The official reasons stated, as far as I can reconstruct them from the explanations given by Cody Gray, are not very extensive. Much more could have been written there and the community could have been included in the process. This is unsatisfying.

Just one thing: If the observation is that decreased rep for upvotes on questions doesn't increase question quality, then one still cannot conclude without doubt that increased rep will not decrease question quality. Maybe it would have been much worse with a different rep gain.

All in all I don't like it.

6
  • Good answers are more valuable than good questions. Do you know what "cause and effect" means?
    – scopchanov
    Nov 14, 2019 at 13:55
  • 5
    @scopchanov "Do you know what "cause and effect" means?" Yes. Nov 14, 2019 at 14:04
  • Good questions help thousands of people googling the same problem. Helping more people makes good answers better, right? Nov 19, 2019 at 4:37
  • @user3226167 Good questions help only indirectly via the good answers. I didn't write that good questions are without value at all. Nov 19, 2019 at 6:50
  • @Trilarion I just want to point out googling is how most people use SO. They do not ask questions. Questions and answers can't be reached have no value to them. I think questions and answers helping more people should get more votes. Nov 20, 2019 at 5:39
  • @user3226167 "...questions and answers helping more people should get more votes." That's definitely the case, if you correlate question and answer scores with number of visits you will get a very high correlation coefficient, I'm quite sure. Nov 20, 2019 at 9:12
36

Thousands of points for a single sentence?

In general I agree: A good question is worth the same reward.

But I doubt the voters motivations…

– Often a vote is given by answeres, just to trigger the OP to come back to vote and accept their answer.
(Remark: In a comment jpmc26 states, that this is not true. It's just my anecdotal observation. And I even read a tutorial about "how to earn reputation" written by somebody suggesting exactly the above and one user telling in a comment, that this tip was extremly helpful. But I cannot find it at the moment...)
– Often a very complicated question has a lot of views, but no votes and no answers. People just don’t understand it. Someone provides a long and comprehensive answer. The OP is excited, millions of thanks, but no extra votes.
Most votes we find at beginner questions. Even if the quality is low. That’s simply a mass effect
Many beginner questions are duplicates but will be answered with a lot of votes (class-room round trips).
Most votes we find on OLD questions with out-dated answers. Often enough a question is closed “as duplicate” with a link to such an old question with a really bad (outdated) accepted answer.
– We can find TOP-Users, gold-badgers at tags, who gave just 3 highly rated contributions.

What I would appreciate:

– A privilege for highly rated users, to mark contributions as “outdated”.
– A privilege for highly rated users, to mark contributions as “excellent”.
– Something like a “community-bounty”, which does not take away the user’s points (in connection with the “excellent” mark?).
– And eventually some “fading away” of old points (in connection with the “outdated” mark?).
– And eventually an upper limit. Earning 1000 points with a single contribution smells a bit.

I've spent many, many hours really earning my current 56k reputation on SO. This change brought ~56k reputation increase to one user who placed a trivial single-sentence question (Look at the last example below...) - without any effort...

Some examples...

To state this first: I really do NOT want to offend anybody here!

This user , was not active for about 8 years. This user has got more than 100k reputation with less than 500 contributions in total. He shows up at the [tsql] tag as "Top-User" with just 3 answers in total. His personal top answer has got ~2400 votes and is absolutely trivial. The question to this answer is very short and has got almost 1700 votes.

One more

This user has got almost 30k reputation with just ~50 contributions. His best question is a trivial one liner with more than 700 ups. His best answer is a trivial code-only answer worth more than 1000 ups. As the underlying question (a trivial one-liner worth almost 600 ups itself) is tagged with [xml], this high answer pushes the user into the [xml] top user list, without even being XML related...

One more

This question is absolutely trivial and has got 4.5k ups. 45k rep just with one single sentence? The user behind is settled at ~50k, so 90% of his total rep is this one single line... The highest answer, (which is not bad!) returned 60k rep to the user.

And really extrem:

The question What are the differences between git pull and git fetch? paid almost 115k(!!!) reputation. Just one single sentence and most trivial content...

This cannot be the way it should be...

2
  • 9
    I disagree with your very first statement, but I've upvoted your answer since I heartily agree with almost all of the rest. I especially like the idea of a total reputation limit on a single post. Thank you.
    – jpmc26
    Nov 19, 2019 at 22:07
  • @jpmc26 Thx for the backup. I just added some examples
    – Shnugo
    Nov 20, 2019 at 10:08
34

One thing that may have been overlooked when it was decided to recalc retrospectively is that working through the lower rep ranges is actually kind of fun.

You would get a little notification, then read about your new privilege and get to try it out. And then strive towards the next one, which was always spaced a little further away than last time - a bit more of a challenge! That was probably a big part of the whole appeal of this site in the first place, and inspired people to stick around and contribute.

I'm not sure how I'd feel about being artificially "boosted" through a whole bunch of privileges suddenly like this, maybe a little bit robbed of the goal I was working towards. I just got boosted up and through 200k and, even though any new privileges are long gone, still felt a little twinge of annoyance.

2
  • 1
    I'm pretty sure The Powers That Be know about gamification and how it works. ;) Nov 13, 2019 at 23:45
  • 8
    @AnsgarWiechers - doubtful. Powers that be are of an ideological persuasion of "participation medals are better than earned achievements", to go on a bumper sticker level of nuance.
    – DVK
    Nov 16, 2019 at 4:11
30

Answers are worth more because they actually provide information. Questions are just shells to be filled with that information. The same question with different information each time isn't worth what the same answer with different information each time is.

People writing answers have to know or find the information they're giving. People writing questions just have to know how to convey their problem clearly.

Hey, just to let you know, I will greatly benefit from this. That doesn't mean I have to like it.

3
  • 9
    One of the main reasons why people end up asking bad questions is because they don’t realize that questions, too, must provide information. I do understand your larger point that there seems to be more useful information conveyed by answers, and that makes them uniquely valuable, but two things: (1) voting always has skewed heavily towards answers, and this change won’t affect that (so answerers will still end up netting more rep), and (2) questions are as necessary as answers to achieve our goal of building a library of high-quality Q&A about programming. Nov 13, 2019 at 21:21
  • 12
    @CodyGray "questions are as necessary as answers" Yes, but I would say that we don't have a shortage of questions really, a shortage of answers is more likely. Nov 14, 2019 at 14:03
  • 6
    @CodyGray Good questions are indeed hard to write, but a lot of those questions have only become good after substantial editing and/or discussions in comments with those knowledgeable in the field. I'm not sure those questions turn into "Community Wiki Questions" (I'm not even sure if those exist). I'm not sure this change gives the appropriate reward to the right people. In addition, the best reward someone asking a question can have is to get a useful answer.
    – Bruno
    Nov 18, 2019 at 16:01
26

I think beyond the hazard of a major recalc granting sweeping permissions to vampire users, this change additionally makes it way easier to game the system for reputation.

Effectively, as long as you can consistently produce questions that gain a down-to-upvote ratio better than 5:1 you stand to farm reputation. What on earth is the community trying to say with that? That nobody deserves to lose reputation without more than an 83% majority vote that they've produced garbage? And that they deserve a reward for anything better than that ludicrously low bar of achievement?

This is only going to encourage people with bad questions to keep coming back and to not improve their behaviour. Why bother when you get rep no matter what?

6
  • 3
    I saw (1) a user post a question with gibberish in the body, (2) it got downvoted to -9 and closed, (3) the user came back and said they accidentally posted too soon and updated the question with a real, albeit simple, question, (4) they got 5 upvotes and multiple votes to reopen the question. Whether or not they intended to game the system or not, they gained 32 reputation from the snafu. :-/ Nov 18, 2019 at 18:50
  • 1
    When they could have posted it right on the first place and gain 50 points instead. Sarcasm mode on Yep. That is a real world manipulation right there. Sarcasm mode off
    – scopchanov
    Nov 19, 2019 at 13:27
  • 3
    @scopchanov The point is there's no incentive. People can just post a bad question every day, pulling a -20/+5 and still make 10 points a day. The worse it is, the more attention it will get, and the more rep, even if it's a really bad question.
    – J...
    Nov 19, 2019 at 13:33
  • People can just post a bad question every day, pulling a -20/+5 and still make 10 points a day => Where do the +5 come (on a bad question).
    – scopchanov
    Nov 19, 2019 at 16:08
  • 4
    @scopchanov I've given up on our side discussion so I'm just going to go ahead and make the point I think you're trying to make, which appears to be expressing your belief that a 'bad' question cannot possibly accumulate any upvotes while remaining a bad question. If that's your belief, then it would seem that you are well aligned to the new policy.
    – J...
    Nov 19, 2019 at 22:43
  • I wrote a response to the story told by @LittleBobbyTables, because I have failed (and I still do) to see the benefit from this particular strategy, if the goal were to harvest r.p. You gave another example, which is fundamentally different, because it doesn't involve any post imrpovement. If you expect me to share my opinion, I need to know what you think the source of those positive votes is, as I try to make no assumptions. As for the new policy, I don't know if I am well aligned with it. Though, I certainly do agree with the "good questions are as valuable as the good answers" part of it.
    – scopchanov
    Nov 20, 2019 at 0:41
24

Can we see some (more) data? Is there more?

In your linked answer, you say it was a "data-led re-evaluation". You also imply that Meta users might care more about this data, but still don't seem to show much of the data.

You link to this answer to justify that the 5-rep-per-question-upvote decision wasn't haven't the desired effect, but this only seems to show that our question quality isn't that great currently, and there's been a decline over time (which could've been caused by any combination of so many factors). It doesn't in any way show the direct effect of the decision. It might be that the decision helped significantly and the site would've been in a much worse state without it. Also, one of the primary metrics used to define a bad question is it having a negative score - this is definitely questionable to use for evaluating this decision, since upvotes counting more would arguably not make much of a difference to questions that aren't upvoted in the first place. Askers of such questions might be motivated to try to avoid downvotes or bans instead of getting upvotes. I would expect it to make more of a difference to users that consistently get an upvote here or there to their questions, but never too many. They'd be much more motivated to try to ask better questions to increase the number of upvotes they get.

It also seems that you're aware we have too many low-quality questions, and this decision was to try to address that. Yet it seems like you were pretty rash in irrevocably reversing it and thus making possibly by far the biggest change to the site in a long time. I say "irrevocably" because of course you could just undo it, but that would greatly upset the people you were trying to appease in the first place by reverting it. This is especially true because it would take some time to get meaningful data that might tell you reverting it was a terrible idea, at which point everyone would be very much used to the new way things are.

I would agree that asking a good question is (or at least can be) hard, but I would need some evidence to believe people asking good questions care much about reputation, especially compared to people asking low-quality questions. I would also be sceptical that reputation hunters would focus on questions instead of answers (and whether they'd even be able to ask a good question), since it's generally the roll of the dice whether a question will be seen by anyone, and there's certainly an element of luck in terms of how it will be received. Not to mention that you actually need to have a decent question before you can ask it (a question that's just another duplicate is not decent, IMO). Although users might be more inclined to ask unusual questions to try to get to HNQ, which can seemingly be done fairly easily in some cases. You may consider this a good or bad thing. I personally consider such questions to be the scourge of the few sites I frequent, thus an increase in such questions would be quite bad, but maybe that's just me.

What would I like to see (or have seen)?

Analysis of what happened following the initial decision.

Consider:

  • Total number of questions
  • Total number of answers
  • Average number of questions and answers per user
  • Scores for questions and answers (the total, average per post and average per user)
  • Other measures of quality (views, closure, deletion)
  • Increases of average post score and quality for individual users over time (i.e. are subsequent questions asked by users better?)

Consider both what happened immediately, and over the next year or so. Include a detailed analysis of the data. Identify and show points in time where other changes occurred which might also affect this data.

Of course this happened during the early days, so it might not be that meaningful and all of that data might not even exist.

Although I hope to at least see the above analysis of the effect of reverting the decision a few months or a year from now.

Some user study or survey to determine what motivates people to (a) ask questions and (b) take time to improve their question before asking.

My primary motivators to ask or improve a question are getting an answer, extremely high standards, contributing to the body of knowledge that is the internet and what others think of me. Reputation is so far down in my list of concerns, it's not even funny. For answers reputation might be a more significant factor. Although that's just me, and I've just asked 6 or so questions on Stack Overflow. It's not that I know everything, it's just that I can almost always find the answer somewhere online.

Trying to check the correlation between score and quality, instead of using score as a measure of quality.

I would focus on those around 0 score (say +3 to -3), as highly upvoted or highly downvoted posts have a much higher chance to be correlated with the other measures of quality.

Other ways to measure quality would include whether it's closed and deleted and the number of views it gets (especially after the initial spike and those coming from search engines). Although, unfortunately, all of these are at least somewhat linked to score.

This would show you whether people are upvoting low quality posts, which would show that this change is likely to encourage more such posts, i.e. do harm.

Check upvotes on low-quality posts.

This is similar to the above point, but instead of looking at score, look at just upvotes.

Here you can also include score as a measure of quality, and you can also include highly downvoted posts.

This would be a stronger signal of whether people are upvoting "low quality" posts.

Analyse other (non-SE) sites.

I wouldn't quite know whether this is ethical, nor what you'd be willing to share publicly about any such analysis, but how other sites are getting by using certain models (at least based on what can be seen publicly) might be a decent measure of which models work and what one might expect when moving in one direction or the other. Even if there are many other differences between SE and those sites.

Some experiments on smaller sites.

The large number of sites on SE could be a great environment for experimentation.

Rolling this out to one or two smaller sites before doing it network-wide would provide some good data for what effects we can expect, and also make undoing it more viable.

On the other hand, having reputation work differently across different sites on the network might be questionable and confusing. Also, sites might not much appreciate being experimented on.

22

A concern I don't see addressed in other answers:

Will this result in more voting rings?

In my opinion, this change incentivizes voting rings.

Consider:

Posting a voting-ring-friendly question is more trivial than an answer:

  1. It doesn't require a relevant question that already exists.
  2. It doesn't have to pass many of the flag checks (e.g. NAA) available on answers

It doubles the reputation for questions, which are more trivial to post than answers

Ergo, the incentive to create a voting ring is increased, while the barriers to operating have been lowered.

1
  • 9
    You raise a good point about voting rings. I'm not sure I'd go so far as to say "incentivizes", but it definitely makes voting rings easier to spin up. Fortunately, us moderators do a pretty decent job of stamping these out. We have functional tools for detecting voting rings, and we are pretty vigilant in enforcing the rules surrounding the sanctity of voting. (Among the moderators who recently resigned were some of the best in the business at dealing with vote fraud, so that's a loss we definitely feel, but we'll continue to do our best in reining these in.) Nov 14, 2019 at 23:30
21

Why only ten?

A good, non-duplicate, on-topic, following the guidelines question is such a rare sight, that in my eyes it is worth a hundred points. No sarcasm.

For me, asking a question is a helluva job. I spend from 30 minutes to several hours in preparations, googling, trying found answers, googling, gathering evidence, googling, double-checking premises, googling, hunting down human errors, googling, creating an MCVE, and googling again. As a result, most questions remain unasked or hastily deleted. Given a question asked with such an effort, with an answer that would serve the community, ten points/vote is too small. We should reward such questions much more.

However. Most questions asked on Stack Overflow do not follow that painful routine. As a matter of fact, most questions a programmer could ask the community are duplicates by definition. Therefore, the voting system should encourage the knowledge reuse, not starting new posts. Imagine Wikipedia rewarding creation of new articles that duplicate existing ones. Imagine Wikipedia that has hundreds of thousands articles on the same topic? That's Stack Overflow.

And now we want to encourage the creation of duplicated articles even more.

2
  • 7
    Implementing bounty system for rewarding good questions seems like better solution than rewarding just about everything.
    – Dalija Prasnikar Mod
    Nov 20, 2019 at 12:41
  • 1
    Re "encourage the knowledge reuse, not starting new posts": Indeed it should. Unfortunately, the system is only optimised (gamified) for providing very quick answers to (new) questions. We are now at 18,565,521 questions. Nov 24, 2019 at 14:36
21

My only issue is the obvious one: for some reason, on Stack Overflow in the last year or so, a lot of poor-quality questions seem to attract an immediate upvote. I can't explain why this has started happening—and perhaps I'm just imagining it—but this new change will obviously lead to disproportionate rewards for users asking such questions.

I still support the proposal, despite the above misgivings. Rewarding even extremely poor-quality questions will encourage more investment in Stack Overflow by the askers, hopefully ultimately resulting in better questions.

... Four Years Later (intoned by the SpongeBob narrator)...

I don't know what I was thinking but the previous paragraph is nonsense. Rewarding low quality questions just results in more low quality questions.

7
  • 27
    "Rewarding even s##t questions will encourage more investment in SO by the askers, hopefully ultimately resulting in better questions." -- hrmm, that's a real slippery slope of an argument. Nov 13, 2019 at 20:40
  • 7
    This has been the case for as long as SO has existed, reenforced by the fact that you earn the ability to upvote first and all of the wording around downvotes discourages you from using them.
    – Kevin B
    Nov 13, 2019 at 20:47
  • @MarcoBonelli: I might call it wishful thinking. Nov 13, 2019 at 21:03
  • 5
    I see this too and the effect now will be that bad question getting upvotes is an even more attractive thing. Just pose your bad question because it will generate rep anyway. We may think this move increases the signal, but instead it may just increase noise. Nov 14, 2019 at 14:29
  • FWIW, I mentioned a couple of reasons why low quality answers may get upvotes here, some of those reasons apply to questions too.
    – PM 2Ring
    Nov 15, 2019 at 22:25
  • @PM2Ring: Some of them are relevant, but I'm seeing shi....err... very poor-quality questions only a few minutes old getting an upvote. Nov 15, 2019 at 23:57
  • 1
    Understood. IME, it mainly happens on homework questions, so I assume it's school friends upvoting each other's posts (and as I said in that answer, they possibly don't even realise they are doing anything wrong). If a few friends are studying together, and they're stuck on their homework, one posts a question, the others upvote it, and they all get the benefit of the answers.
    – PM 2Ring
    Nov 16, 2019 at 0:33
19

I don't overestimate the value of internet points, but they do have some value. If they didn't, no one would want to do anything to get more of them. The premise of Stack Overflow is gamification.

From some material quoted in one of Jeff Atwood's blog posts

What we saw as our membership scrabbled over valueless points was that there didn't actually need to be any sort of material reward other than the points themselves. We didn't need to allow them to trade the points in for benefits, virtual or otherwise. It was enough of a reward for most people just to see their points wobble upwards.

We shouldn't care that a whole ton of extra points are suddenly thrown around. We shouldn't care that in many new cases questions will be worth more points than their answers. So what? They don't mean anything. But by that same logic, we shouldn't have cared about earning points with our answers in the first place.

This change isn't horrible (or at least I'm not in a hurry to jump to that conclusion.) Like many, I don't have really strong feelings about it. But to whatever extent points did represent knowledge or expertise (factoring in that there were already points for questions) they will represent it a little less.

Another reason why it feels a bit wrong is the timing. Stack Exchange has damaged their reputation by engaging in what many consider unethical behavior aimed at members of their own community. Aside from that they've shown an interest in improving their reputation. It feels like they're trying to minimize the harm and just make friends by printing a whole ton of those points we work for. Giving something away devalues it. Giving it away to create a more favorable impression of the company makes it sting.

It feels small saying that. They're just meaningless internet points. I also answer questions just because I like helping people. Reading and answering questions has benefited me in many significant ways. Perhaps that's why I don't feel more strongly. But even if the value is arbitrary, it motivates people, and this makes that value a little bit cheaper.

6
  • 10
    Well, not entirely useless. Enough reputation = privileges. All depends on what people who gain privileges do with them, of course... Nov 13, 2019 at 21:04
  • 2
    It was once sold as a way to easily be hired (it is after all a single number that a machine can easily process). But HR persons and recruiters probably very quickly realised that it was being gamed. Nov 13, 2019 at 21:46
  • 1
    @PeterMortensen - It still carries some weight. If you've got some rep potential employers pay some attention. They'll at least look at your answers. Nov 13, 2019 at 22:33
  • 5
    @CindyMeister - Many of what we call "privileges" are just extra ways to donate time to the site. I'm not saying that's bad, but it's a weird sort of privilege. Getting to relax in airline lounge because you have lots of miles is a privilege. Getting to be the flight attendant and walk around telling people to put their seats in the upright position, not so much. (Getting to fly the plane would be cool.) Nov 14, 2019 at 15:47
  • 1
    what's horrible is that it is applied retroactively. that is truly horrible. IMO. and they didn't just change the final value. they changed the whole history of your rep (at stackexchange.com/users/NNN/name?tab=reputation). truly Orwellian.
    – Will Ness
    Nov 14, 2019 at 17:25
  • 2
    @Scott Hannen: You are correct. A real-world example from a few years back (reputation points actually used as a filter): "GitLab ... because I did not have enough points on Stack Overflow, they told me 'no'." It is no wonder voting rings exist. Nov 18, 2019 at 16:20
17

This is about 9 years overdue, and I'm happy for that; but there are a lot more pressing issues to fix first, not the least of which is rebuilding the trust between the community, the moderators, and the company.

If Stack Overflow was publicly working to rebuild trust, I'd be glad to give them kudos on this; but right now this is coming across as a deflection from the problems we are encountering -- a way to mollify us without actually addressing the underlying issues.

11
  • 42
    Rebuild trust by avoiding any discussion a priori ? Rebuild trust by pretending the problem is the whistle-blower and not the decisions made behind walls ? Rebuild trust by ignoring the departure and complaints of users hit with floods of identical bad questions ? Rebuild trust by breaking the contract which made us answer, that is that "reputation" was also a measure of problem solving ability and technical knowledge and had some kind of value ? This is not rebuilding trust, it's just definitely killing it. Nov 13, 2019 at 19:57
  • 17
    I completely understand the sentiment here, and have had similar feelings myself. A couple of things to keep in mind, just as devil's advocate. First, although staff didn't discuss this with the community at large, they did share it with moderators in private channels, and they did listen to our feedback. What was announced today is not the same as what was originally drafted. That's a huge step forward, so it deserves recognition. Second, the lower-level staff who is responsible for implementing the day-to-day business of Q&A is not the same staff who has the authority to rebuild trust. Nov 13, 2019 at 20:00
  • 13
    It's not coming off as a deflection in the process of rebuilding trust to me. It feels like twisting the knife. This was not discussed with the community, not what the community wanted (per this), and not what it needed.
    – Erik A
    Nov 13, 2019 at 20:10
  • 48
    @CodyGray "Second, the lower-level staff who is responsible for implementing the day-to-day business of Q&A is not the same staff who has the authority to rebuild trust." - while I feel this is true, it certainly doesn't help that Sara is the one to make this blog. At this point, there's clear negative sentiment built up based on her involvement with pronoun-gate and fallout. Thus by posting this blog it feels like complete disregard of the aforementioned more pressing issues.
    – Mysticial
    Nov 13, 2019 at 20:19
  • 9
    @DenysSéguret That's exactly the sentiment that me and probably a lot of other users are having right now. You should really post your comment as an answer. Nov 13, 2019 at 20:26
  • @MarcoBonelli I tried to make an answer of the main point Nov 13, 2019 at 20:44
  • 2
    Hey, people who are downvoting: Read the whole thing. It's not all evil.
    – S.S. Anne
    Nov 13, 2019 at 20:51
  • 10
    @JL2210, yes, but we can't split our votes between the first and second paragraphs. Both paragraphs are relevant, but I voted on the paragraph that's more relevant. Nov 13, 2019 at 21:12
  • 1
    @ScottHannen Most of the first paragraph is even saying not now.
    – S.S. Anne
    Nov 13, 2019 at 21:13
  • 26
    "What was announced today is not the same as what was originally drafted. That's a huge step forward" Saddest thing I've heard in 2019
    – Unihedron
    Nov 13, 2019 at 21:29
  • 3
    @Mysticial It's more the fact that she is demonstrably an ideologue who rejects the premises upon which SO was built (particularly the premises of valuing your dedicated users, treating them with respect even if you disagree with them, and prioritizing content quality over personal feelings). Given how different her views are, it's not clear why she would want to work at SO, other than to use its success as a starting point for her own ideal rather than start fresh somewhere else. Some people earn your trust with the pattern of their actions, and others lose it. Sara is in the latter category.
    – jpmc26
    Nov 17, 2019 at 18:37
15

I would be super curious to know if there is any downward trend in questions receiving upvotes after this change.

I've been lurking on the "Newest" tab since this was announced, and I really feel like the number of people upvoting has gone down noticeably. It could just be confirmation bias, but I feel like people are being even stingier with their upvotes.

2
  • 8
    Heh. Well, that certainly would be an ironic side-effect. That data should be accessible from SEDE after a couple of weeks. Nov 15, 2019 at 20:38
  • 15
    @LittleBobbyTables I've certainly been freer with my downvotes since the change (since they count for some much less now). That's not quite the effect you suggest out, but similarly I suspect it isn't the effect they're after.
    – DavidW
    Nov 16, 2019 at 14:33
15

Re-iterating what so many others have said:

This is a horrible move. It enables help vampires (and notoric bad-question-askers in general) and devalues the work of people who contribute by answering questions.

Secondly, it is yet another "who cares what the community feels" move. Community feedback - bah! L’État, c’est moi!

Personally, I've had enough. Starting today I'll cease all contributions to the Stack Exchange network - no more reviewing, no more edits, no more flagging, no more answers. If I need to ask a question to help myself, sure I'll do that (and hey, I'll get 10 rep per upvote), but that's it.

You must log in to answer this question.

Not the answer you're looking for? Browse other questions tagged .